www.vorhilfe.de
- Förderverein -
Der Förderverein.

Gemeinnütziger Verein zur Finanzierung des Projekts Vorhilfe.de.
Hallo Gast!einloggen | registrieren ]
Startseite · Mitglieder · Impressum
Forenbaum
^ Forenbaum
Status VH e.V.
  Status Vereinsforum

Gezeigt werden alle Foren bis zur Tiefe 2

Navigation
 Startseite...
 Suchen
 Impressum
Das Projekt
Server und Internetanbindung werden durch Spenden finanziert.
Organisiert wird das Projekt von unserem Koordinatorenteam.
Hunderte Mitglieder helfen ehrenamtlich in unseren moderierten Foren.
Anbieter der Seite ist der gemeinnützige Verein "Vorhilfe.de e.V.".
Partnerseiten
Weitere Fächer:

Open Source FunktionenplotterFunkyPlot: Kostenloser und quelloffener Funktionenplotter für Linux und andere Betriebssysteme
Forum "Funktionen" - Maximaler Abstand
Maximaler Abstand < Funktionen < eindimensional < reell < Analysis < Hochschule < Mathe < Vorhilfe
Ansicht: [ geschachtelt ] | ^ Forum "Funktionen"  | ^^ Alle Foren  | ^ Forenbaum  | Materialien

Maximaler Abstand: Frage (beantwortet)
Status: (Frage) beantwortet Status 
Datum: 22:01 Di 19.06.2012
Autor: hilbert

Hallo Leute,

ich soll den maximalen Abstand der Fläche [mm] x^3+2x^2-2x+y^2=0 [/mm] vom Ursprung bestimmen. Dazu muss ich doch die Abstandsfunktion d(x,y) bestimmen und diese maximieren oder? Eigentlich suche ich doch einen Punkt (x,y) der auf dieser Fläche liegt, sodass [mm] ||(x,y)-(0,0)||_2 [/mm] am größten ist.

Soll ich jetzt einfach [mm] x^3+2x^2-2x+y^2=0 [/mm] nach y umstellen und für x einsetzen oder wie ist das gemeint?

Schonmal vielen Dank

        
Bezug
Maximaler Abstand: Antwort
Status: (Antwort) fertig Status 
Datum: 22:18 Di 19.06.2012
Autor: MathePower

Hallo hilbert,



> Hallo Leute,
>  
> ich soll den maximalen Abstand der Fläche
> [mm]x^3+2x^2-2x+y^2=0[/mm] vom Ursprung bestimmen. Dazu muss ich
> doch die Abstandsfunktion d(x,y) bestimmen und diese
> maximieren oder? Eigentlich suche ich doch einen Punkt
> (x,y) der auf dieser Fläche liegt, sodass
> [mm]||(x,y)-(0,0)||_2[/mm] am größten ist.
>
> Soll ich jetzt einfach [mm]x^3+2x^2-2x+y^2=0[/mm] nach y umstellen
> und für x einsetzen oder wie ist das gemeint?
>  


Das ist eine Aufgabe, die mit der Mulipikatorenmethode
nach Lagrange gelöst werden kann.

Dazu ist die Abstandsfunktion zweier Punkte unter der
Nebenbedingung, daß der gesuchte Punkt auf der gegebenen
Fläche liegt, zu maximieren.


> Schonmal vielen Dank


Gruss
MathePower

Bezug
                
Bezug
Maximaler Abstand: Frage (beantwortet)
Status: (Frage) beantwortet Status 
Datum: 22:28 Di 19.06.2012
Autor: hilbert

Hallo Mathepower,

also habe ich die Abstandsfunktion [mm] d(x,y)=\sqrt{x^2+y^2} [/mm] da mein anderer Punkt ja der 0 Punkt ist. Meine Nebenbedingung ist, dass [mm] x^3+2x^2-2x+y^2=0 [/mm] ist.

Dann habe ich [mm] L(x,y,\lambda)=\sqrt{x^2+y^2}+\lambda(x^3+2x^2-2x+y^2). [/mm] Mit den partiellen Ableitungen:

[mm] \bruch{\partial L}{\partial x} [/mm] = [mm] \bruch{2x}{2\sqrt{x^2+y^2}}+\lambda(3x^2+4x+2) [/mm] = 0
[mm] \bruch{\partial L}{\partial y} [/mm] = [mm] \bruch{2y}{2\sqrt{x^2+y^2}}+2y\lambda [/mm] = 0
[mm] \bruch{\partial L}{\partial \lambda} [/mm] = [mm] x^3+2x^2-2x+y^2=0. [/mm]

Und die Aufgabe ist nun, dieses Gleichungssystem zu lösen?
Oder habe ich bis hier hin schon einen Fehler gemacht?

Schonmal Danke!

Bezug
                        
Bezug
Maximaler Abstand: Antwort
Status: (Antwort) fertig Status 
Datum: 22:40 Di 19.06.2012
Autor: MathePower

Hallo hilbert,

> Hallo Mathepower,
>  
> also habe ich die Abstandsfunktion [mm]d(x,y)=\sqrt{x^2+y^2}[/mm] da
> mein anderer Punkt ja der 0 Punkt ist. Meine Nebenbedingung
> ist, dass [mm]x^3+2x^2-2x+y^2=0[/mm] ist.
>  
> Dann habe ich
> [mm]L(x,y,\lambda)=\sqrt{x^2+y^2}+\lambda(x^3+2x^2-2x+y^2).[/mm] Mit
> den partiellen Ableitungen:
>  
> [mm]\bruch{\partial L}{\partial x}[/mm] =
> [mm]\bruch{2x}{2\sqrt{x^2+y^2}}+\lambda(3x^2+4x+2)[/mm] = 0
>  [mm]\bruch{\partial L}{\partial y}[/mm] =
> [mm]\bruch{2y}{2\sqrt{x^2+y^2}}+2y\lambda[/mm] = 0
>  [mm]\bruch{\partial L}{\partial \lambda}[/mm] = [mm]x^3+2x^2-2x+y^2=0.[/mm]
>  
> Und die Aufgabe ist nun, dieses Gleichungssystem zu
> lösen?
>  Oder habe ich bis hier hin schon einen Fehler gemacht?
>  


Einen Fehler im Sinne, dass die Abstandsfunktion
ungeschickt gewählt wurde.

Wähle als Abstandsfunktion

[mm]d(x,y)=x^2+y^2[/mm]


> Schonmal Danke!


Gruss
MathePower

Bezug
                                
Bezug
Maximaler Abstand: Frage (beantwortet)
Status: (Frage) beantwortet Status 
Datum: 22:49 Di 19.06.2012
Autor: hilbert

Okay, da der Abstand immer positiv ist, macht die so gewählte Abstandsfunktion auch Sinn.

Dann käme ich auf [mm] L(x,y,\lambda)=x^2+y^2+\lambda(x^3+2x^2-2x+y^2) [/mm]

und die Ableitungen sind dann:

[mm] 2x+\lambda(3x^2+4x-2)=0 [/mm]

[mm] 2y+2y\lambda [/mm] = 0

und
[mm] x^3+2x^2-2x+y^2=0 [/mm]


Aus der 2. Gleichung folgt [mm] 2y(1+\lambda)=0, [/mm] also ich y = 0 oder [mm] \lambda [/mm] = -1. Für y = 0 kann ich dann [mm] x^3+2x^2-2x=0 [/mm] lösen.

Für [mm] \lambda [/mm] = -1
Habe ich dann in der ersten Gleichung [mm] 2x-3x^2+4x-2=0 [/mm] was ich lösen kann. Und mit der Lösung und der Nebenbedingung kann ich dann y ausrechnen.

Wäre das dann die richtige Lösung?

Vielen Dank

Bezug
                                        
Bezug
Maximaler Abstand: Mitteilung
Status: (Mitteilung) Reaktion unnötig Status 
Datum: 00:44 Mi 20.06.2012
Autor: Marcel

Hallo,

> Okay, da der Abstand immer positiv ist, macht die so
> gewählte Abstandsfunktion auch Sinn.

na, die Sprechweise ist eher weniger sinnvoll. Es gibt eigentlich weniger "den Abstand" als vielmehr "einen Abstand". Denn das ist der Sinn einer Funktion namens "Metrik" - und i.a. hat man sehr viele Möglichkeiten, Metriken zu wählen.

In der Tat nehmen Metriken stets Werte [mm] $\ge [/mm] 0$ an. Aber obige Funktion ist keine Metrik im Sinne der Definition des Begriffes "Metrik". Es ist sowieso auch nicht direkt eine Metrik, auch Dein [mm] $d(x,y)=\sqrt{x^2+y^2}\,$ [/mm] ist dies nicht. Eine "Abstandsfunktion" im [mm] $\IR^2$ [/mm] mißt den Abstand zwischen zwei Punkten des [mm] $\IR^2$ [/mm] - das machst Du auch, schreibst aber nirgendswo den zweiten Punkt, nämlich [mm] $(0,0)\,,$ [/mm] dazu. Klar: Warum auch? Den halten wir eh fest! (Das macht man bei einer "Abstandsfunktion" im [mm] $\IR^2$ [/mm] aber nicht - dort wird ja nicht ein Punkt fixiert, sondern beide sind variabel. ABER: Man kann auch eine Abstandsfunktion im [mm] $\IR^2$ [/mm] betrachten, wo man "den Abstand eines Punktes zu einer Teilmenge des [mm] $\IR^2$" [/mm] mißt. So etwas wäre Deine Abstandsfunktion, die Menge wäre einfach die einelementige Teilmenge [mm] $\{(0,0)\} \subseteq \IR^2\,.$ [/mm] Aber das nur nebenbei...)
Vielleicht arbeitest Du besser mit dem Begriff "Norm" - denn was ist im euklidischen [mm] $\IR^2$ [/mm] auch "im geometrischen Sinne" der Abstand eines Punktes zum Nullpunkt?

Und warum Mathepower gesagt hat: "Bei der 'Abstandsfunktion' wähle doch besser einfach [mm] $\|(x,y)\|^2$" [/mm]

Da liegt z.B. die einfache folgende Überlegung zugrunde: Ist $f: M [mm] \to [0,\infty)\,,$ [/mm] so hat [mm] $f\,$ [/mm] genau dann an [mm] $m_0 \in [/mm] M$ ein Maximum, wenn [mm] $g:=f^2$ [/mm] an [mm] $m_0$ [/mm] ein Maximum hat.

Warum? Im Wesentlichen, weil etwa $q: [mm] [0,\infty) \to [0,\infty)$ [/mm] mit [mm] $q(x):=x^2$ [/mm] ($x [mm] \ge [/mm] 0$) streng wächst. (Dass Deine Begründung zu wenig ist, siehst Du etwa, wenn Du mal überlegst, was gewesen wäre, wenn Du 'als Abstandsfunktion' (bzgl. $(0,0)$) die Funktion [mm] $f((x,y)):=|\sin(\sqrt{x^2+y^2})|$ [/mm] genommen hättest. Das hätte Dir nichts gebracht, obwohl dieses [mm] $f\,$ [/mm] auch nur Werte [mm] $\ge [/mm] 0$ annimmt. Nur: Die nachgeschaltete Funktion [mm] $|\sin(.)|$ [/mm] ist halt nicht mehr streng monoton wachsend...)

Gruß,
  Marcel

Bezug
                                                
Bezug
Maximaler Abstand: Mitteilung
Status: (Mitteilung) Reaktion unnötig Status 
Datum: 01:19 Mi 20.06.2012
Autor: scherzkrapferl

Hallo Marcel,

> Und warum Mathepower gesagt hat: "Bei der
> 'Abstandsfunktion' wähle doch besser einfach [mm]\|(x,y)\|^2[/mm]"
>  
> Da liegt z.B. die einfache folgende Überlegung zugrunde:
> Ist [mm]f: M \to [0,\infty)\,,[/mm] so hat [mm]f\,[/mm] genau dann an [mm]m_0 \in M[/mm]
> ein Maximum, wenn [mm]g:=f^2[/mm] an [mm]m_0[/mm] ein Maximum hat.
>  
> Warum? Im Wesentlichen, weil etwa [mm]q: [0,\infty) \to [0,\infty)[/mm]
> mit [mm]q(x):=x^2[/mm] ([mm]x \ge 0[/mm]) streng wächst.
>  
> Gruß,
>    Marcel

hab mal grad eben so drübergelesen. Was vielleicht zu erwähnen wäre (nicht jedem studenten/schüler klar, teilweise auch verwirrend) ist dass man für den ansatz [mm] $\phi=x^{2}+y^{2}=0$ [/mm] zum schluss noch die wuzel ziehen muss um auf das gewünschte ergebnis zu kommen (zumindest wurde das bei mir so gehandhabt). hatte mal einen kollegen der alles richtig gemacht hat, doch zum schluss vergessen hat die wurzel zu ziehen und deshalb die prüfung nicht geschaft hat (bsp: abstand vom punkt (0,0) zb ...).

das mit dem "wurzelziehen" beziehe ich allerdings auf beispiele bei denen nach dem abstand von einem punkt (zb: (0,0)) gefragt wurde. beim allgemeinen fall kenne ich mich leider nicht so gut aus (das leid eines physikers).

würde mich interessieren ob meine annahme auch für den allgemeinen fall gilt :) vielleicht kannst du mir ja nur kurz mit "ja" oder "nein" antworten ^^ ..


LG und gute Nacht,
Scherzkrapferl


Bezug
                                                        
Bezug
Maximaler Abstand: Unterschied ExtremSTELLE/WERT
Status: (Mitteilung) Reaktion unnötig Status 
Datum: 08:07 Mi 20.06.2012
Autor: Marcel

Hallo Krapferl,

> Hallo Marcel,
>  
> > Und warum Mathepower gesagt hat: "Bei der
> > 'Abstandsfunktion' wähle doch besser einfach [mm]\|(x,y)\|^2[/mm]"
>  >  
> > Da liegt z.B. die einfache folgende Überlegung zugrunde:
> > Ist [mm]f: M \to [0,\infty)\,,[/mm] so hat [mm]f\,[/mm] genau dann an [mm]m_0 \in M[/mm]
> > ein Maximum, wenn [mm]g:=f^2[/mm] an [mm]m_0[/mm] ein Maximum hat.
>  >  
> > Warum? Im Wesentlichen, weil etwa [mm]q: [0,\infty) \to [0,\infty)[/mm]
> > mit [mm]q(x):=x^2[/mm] ([mm]x \ge 0[/mm]) streng wächst.
>  >  
> > Gruß,
>  >    Marcel
>
> hab mal grad eben so drübergelesen. Was vielleicht zu
> erwähnen wäre (nicht jedem studenten/schüler klar,
> teilweise auch verwirrend) ist dass man für den ansatz
> [mm]\phi=x^{2}+y^{2}=0[/mm] zum schluss noch die wuzel ziehen muss
> um auf das gewünschte ergebnis zu kommen (zumindest wurde
> das bei mir so gehandhabt). hatte mal einen kollegen der
> alles richtig gemacht hat, doch zum schluss vergessen hat
> die wurzel zu ziehen und deshalb die prüfung nicht
> geschaft hat (bsp: abstand vom punkt (0,0) zb ...).
>
> das mit dem "wurzelziehen" beziehe ich allerdings auf
> beispiele bei denen nach dem abstand von einem punkt (zb:
> (0,0)) gefragt wurde. beim allgemeinen fall kenne ich mich
> leider nicht so gut aus (das leid eines physikers).
>
> würde mich interessieren ob meine annahme auch für den
> allgemeinen fall gilt :) vielleicht kannst du mir ja nur
> kurz mit "ja" oder "nein" antworten ^^ ..

na, pass' auf: Die "Wurzel ziehen" musst Du, wenn Du den konkreten Abstand berechnen willst - das ist ja sozusagen hier der Dich interessierende "Funktionswert".

Vielleicht erkläre ich es mal anders:
Wir nehmen an, die Funktion $f: M [mm] \to \IR$ [/mm] habe genau eine Maximalstelle [mm] $m_0 \in M\,.$ [/mm] Wenn nun $g: I [mm] \to \IR$ [/mm] streng wächst ($I [mm] \subseteq \IR$ [/mm] soll dabei den Wertebereich von [mm] $f\,$ [/mm] umfassen!), dann hat auch $g [mm] \circ [/mm] f: M [mm] \to \IR$ [/mm] genau eine Maximalstelle, und zwar die gleiche wie [mm] $f\,,$ [/mm] nämlich [mm] $m_0 \in M\,.$ [/mm]

Wenn man nun fragt, wie denn auch der Wert [mm] $f(m_0)$ [/mm] aussieht, so kann man natürlich nicht $(g [mm] \circ f)(m_0)$ [/mm] mit [mm] $f(m_0)$ [/mm] identifizieren - sondern, wenn man $(g [mm] \circ f)(m_0)$ [/mm] kennt, dann kann man zurückrechnen
[mm] $$f(m_0)=g^{-1}(g(f(m_0)))\,.$$ [/mm]

Okay?

Hier war ja quasi $f: [mm] \IR^2 \supseteq [/mm] A [mm] \to \IR$ [/mm] nichts anderes als die euklidische Standardnom eingeschränkt auf irgendeine gewisse Teilmenge [mm] $A\,$ [/mm] des [mm] $\IR^2\,,$ [/mm] die aus der Aufgabenstellung folgt.
Betrachten wir erstmal dieses [mm] $f\,,$ [/mm] d.h. [mm] $\IR^2 \supseteq [/mm] A [mm] \in x=(x_1,x_2) \mapsto f(x)=f((x_1,x_2))=:f(x_1,x_2):=\|x\|_2=\sqrt{x_1^2+x_2^2}\,.$ [/mm]

Okay? Wenn Du nun fragst "Wo wird der maximale Abstand auf [mm] $A\,$ [/mm] zum Nullpunkt $(0,0)$ des [mm] $\IR^2$ [/mm] angenommen?", dann kannst Du versuchen, einen (oder mehrere) Punkt(e) von [mm] $A\,$ [/mm] so zu bestimmen, nennen wir solch' einen [mm] $m_0\,,$ [/mm] so dass [mm] $f\,$ [/mm] an diesem [mm] $m_0 \in [/mm] A$ maximal wird. Das kannst Du auch machen, indem Du irgendeine streng monoton wachsende Funktion $I [mm] \to \IR$ [/mm] nach [mm] $f\,$ [/mm] schaltest ($I [mm] \subseteq \IR$ [/mm] soll den Wertebereich von [mm] $f\,$ [/mm] umfassen), d.h. Du betrachtest beispielsweise die Quadratfunktion $g: [mm] [0,\infty) \to \IR$ [/mm] (der Definitionsbereich von [mm] $g\,$ [/mm] sollte allerdings natürlich den Wertebereich von [mm] $f\,$ [/mm] umfassen!) und bestimmst bzgl. $g [mm] \circ [/mm] f$ dann "Maximalstelle(n)".

Mathepower hat quasi vorgeschlagen, die streng monoton wachsende Funktion [mm] $g:[0,\infty) \to [0,\infty)$ [/mm] "nachzuschalten", wobei das [mm] $f\,$ [/mm] wie oben ist. Er sagt dann:
Anstatt [mm] $f\,$ [/mm] auf Maximalstellen in [mm] $A\,$ [/mm] zu untersuchen, kann man auch $g [mm] \circ f=f^2$ [/mm] auf Maximalstellen untersuchen - denn diese haben die gleichen MAXIMALSTELLEN in [mm] $A\,.$ [/mm]

Der Wert der Maximalstellen ist natürlich ein anderer: Wenn [mm] $m_0 \in [/mm] A$ eine Maximalstelle in [mm] $A\,$ [/mm] für [mm] $f\,$ [/mm] ist, so nimmt [mm] $f\,$ [/mm] dort den Wert [mm] $f(m_0)$ [/mm] an, aber $g [mm] \circ [/mm] f$ natürlich den Wert [mm] $g(f(m_0))\,.$ [/mm]

Bei Dir also: Wenn [mm] $m_0$ [/mm] eine Maximalstelle in [mm] $A\,$ [/mm] bzgl. $f [mm] \,$ [/mm] ist, dann ist [mm] $f(m_0)=\|m_0\|_2=\|m_0\|$ [/mm] "der Abstand von [mm] $m_0$ [/mm] zu $(0,0)$". Mit $(g [mm] \circ [/mm] f)$ (mit [mm] $g(x)=x^2$ [/mm] für $x [mm] \ge [/mm] 0$) findest Du die gleiche Maximalstelle, aber der Funktionswert $(g [mm] \circ f)(m_0)=g(f(m_0))=\|m_0\|^2$ [/mm] gibt natürlich nicht den Abstand von [mm] $m_0$ [/mm] zu $(0,0)$ an, sondern eben das Quadrat dieses Abstands. Daher musst Du, wenn Du [mm] $m_0$ [/mm] kennst, aber [mm] $f(m_0)$ [/mm] berechnen willst, hier bei [mm] $g(f(m_0))$ [/mm] noch [mm] $g^{-1}$ [/mm] nachschalten.

Also das einzige, was Du Dir klar machen musst: Wenn ich hier ein streng wachsende Funktion nach einer anderen schalte, dann nimmt diese Verkettung an genau den gleichen Stellen wie die Ausgangsfunktion ihre Maxima und Minima an (und solche sind bei beiden gleichzeitig existent oder nicht etc. pp.).

Die ExtremSTELLEN sind gleich (und von gleicher Art).

Die Verkettung hat aber natürlich andere FunktionsWERTE, d.h. die ExtremWERTE sind verschieden - aber mit [mm] $g\,$ [/mm] bzw. [mm] $g^{-1}$ [/mm] kann man sie einander in Bezug bringen.

Gruß,
  Marcel

Bezug
                                                                
Bezug
Maximaler Abstand: Mitteilung
Status: (Mitteilung) Reaktion unnötig Status 
Datum: 22:49 Mi 20.06.2012
Autor: scherzkrapferl

Hallo Marcel ;)

> Hallo Krapferl,

> na, pass' auf: Die "Wurzel ziehen" musst Du, wenn Du den
> konkreten Abstand berechnen willst - das ist ja sozusagen
> hier der Dich interessierende "Funktionswert".

Da lag der Hund begraben. Alles klar ^^ Bei uns wurde immer nach konkreten Abständen gefragt.

>  
> Vielleicht erkläre ich es mal anders:
>  Wir nehmen an, die Funktion [mm]f: M \to \IR[/mm] habe genau eine
> Maximalstelle [mm]m_0 \in M\,.[/mm] Wenn nun [mm]g: I \to \IR[/mm] streng
> wächst ([mm]I \subseteq \IR[/mm] soll dabei den Wertebereich von
> [mm]f\,[/mm] umfassen!), dann hat auch [mm]g \circ f: M \to \IR[/mm] genau
> eine Maximalstelle, und zwar die gleiche wie [mm]f\,,[/mm] nämlich
> [mm]m_0 \in M\,.[/mm]
>  
> Wenn man nun fragt, wie denn auch der Wert [mm]f(m_0)[/mm] aussieht,
> so kann man natürlich nicht [mm](g \circ f)(m_0)[/mm] mit [mm]f(m_0)[/mm]
> identifizieren - sondern, wenn man [mm](g \circ f)(m_0)[/mm] kennt,
> dann kann man zurückrechnen
>  [mm]f(m_0)=g^{-1}(g(f(m_0)))\,.[/mm]
>  
> Okay?
>  
> Hier war ja quasi [mm]f: \IR^2 \supseteq A \to \IR[/mm] nichts
> anderes als die euklidische Standardnom eingeschränkt auf
> irgendeine gewisse Teilmenge [mm]A\,[/mm] des [mm]\IR^2\,,[/mm] die aus der
> Aufgabenstellung folgt.
>  Betrachten wir erstmal dieses [mm]f\,,[/mm] d.h. [mm]\IR^2 \supseteq A \in x=(x_1,x_2) \mapsto f(x)=f((x_1,x_2))=:f(x_1,x_2):=\|x\|_2=\sqrt{x_1^2+x_2^2}\,.[/mm]
>  
> Okay? Wenn Du nun fragst "Wo wird der maximale Abstand auf
> [mm]A\,[/mm] zum Nullpunkt [mm](0,0)[/mm] des [mm]\IR^2[/mm] angenommen?", dann kannst
> Du versuchen, einen (oder mehrere) Punkt(e) von [mm]A\,[/mm] so zu
> bestimmen, nennen wir solch' einen [mm]m_0\,,[/mm] so dass [mm]f\,[/mm] an
> diesem [mm]m_0 \in A[/mm] maximal wird. Das kannst Du auch machen,
> indem Du irgendeine streng monoton wachsende Funktion [mm]I \to \IR[/mm]
> nach [mm]f\,[/mm] schaltest ([mm]I \subseteq \IR[/mm] soll den Wertebereich
> von [mm]f\,[/mm] umfassen), d.h. Du betrachtest beispielsweise die
> Quadratfunktion [mm]g: [0,\infty) \to \IR[/mm] (der
> Definitionsbereich von [mm]g\,[/mm] sollte allerdings natürlich den
> Wertebereich von [mm]f\,[/mm] umfassen!) und bestimmst bzgl. [mm]g \circ f[/mm]
> dann "Maximalstelle(n)".
>  
> Mathepower hat quasi vorgeschlagen, die streng monoton
> wachsende Funktion [mm]g:[0,\infty) \to [0,\infty)[/mm]
> "nachzuschalten", wobei das [mm]f\,[/mm] wie oben ist. Er sagt
> dann:
>  Anstatt [mm]f\,[/mm] auf Maximalstellen in [mm]A\,[/mm] zu untersuchen, kann
> man auch [mm]g \circ f=f^2[/mm] auf Maximalstellen untersuchen -
> denn diese haben die gleichen MAXIMALSTELLEN in [mm]A\,.[/mm]
>  
> Der Wert der Maximalstellen ist natürlich ein anderer:
> Wenn [mm]m_0 \in A[/mm] eine Maximalstelle in [mm]A\,[/mm] für [mm]f\,[/mm] ist, so
> nimmt [mm]f\,[/mm] dort den Wert [mm]f(m_0)[/mm] an, aber [mm]g \circ f[/mm]
> natürlich den Wert [mm]g(f(m_0))\,.[/mm]
>  
> Bei Dir also: Wenn [mm]m_0[/mm] eine Maximalstelle in [mm]A\,[/mm] bzgl. [mm]f \,[/mm]
> ist, dann ist [mm]f(m_0)=\|m_0\|_2=\|m_0\|[/mm] "der Abstand von [mm]m_0[/mm]
> zu [mm](0,0)[/mm]". Mit [mm](g \circ f)[/mm] (mit [mm]g(x)=x^2[/mm] für [mm]x \ge 0[/mm])
> findest Du die gleiche Maximalstelle, aber der
> Funktionswert [mm](g \circ f)(m_0)=g(f(m_0))=\|m_0\|^2[/mm] gibt
> natürlich nicht den Abstand von [mm]m_0[/mm] zu [mm](0,0)[/mm] an, sondern
> eben das Quadrat dieses Abstands. Daher musst Du, wenn Du
> [mm]m_0[/mm] kennst, aber [mm]f(m_0)[/mm] berechnen willst, hier bei
> [mm]g(f(m_0))[/mm] noch [mm]g^{-1}[/mm] nachschalten.
>  
> Also das einzige, was Du Dir klar machen musst: Wenn ich
> hier ein streng wachsende Funktion nach einer anderen
> schalte, dann nimmt diese Verkettung an genau den gleichen
> Stellen wie die Ausgangsfunktion ihre Maxima und Minima an
> (und solche sind bei beiden gleichzeitig existent oder
> nicht etc. pp.).
>
> Die ExtremSTELLEN sind gleich (und von gleicher Art).
>  
> Die Verkettung hat aber natürlich andere FunktionsWERTE,
> d.h. die ExtremWERTE sind verschieden - aber mit [mm]g\,[/mm] bzw.
> [mm]g^{-1}[/mm] kann man sie einander in Bezug bringen.
>  

Wow ! Da hast du dir echt viel Mühe gegeben. Habe gerade 20 min gebraucht um das nachzuvollziehen. Klingt allerdings alles sehr schlüssig und nachvollziehbar :) - gefällt. Solltest Lehrer/Dozent werden ;D (falls du das nicht schon bist)

Danke für die ausführliche Erläuterung. Hätte nicht erwartet dass du dir so viel Zeit dafür nimmst :) Respekt.

Hab auf jedenfall was gelernt. THX

> Gruß,
>    Marcel

Beste Grüße,
Scherzkrapferl


Bezug
                                                                        
Bezug
Maximaler Abstand: Mitteilung
Status: (Mitteilung) Reaktion unnötig Status 
Datum: 23:16 Mi 20.06.2012
Autor: Marcel

Hallo,

> Hallo Marcel ;)
>  
> > Hallo Krapferl,
>  
> > na, pass' auf: Die "Wurzel ziehen" musst Du, wenn Du den
> > konkreten Abstand berechnen willst - das ist ja sozusagen
> > hier der Dich interessierende "Funktionswert".
>  
> Da lag der Hund begraben. Alles klar ^^ Bei uns wurde immer
> nach konkreten Abständen gefragt.
>  
> >  

> > Vielleicht erkläre ich es mal anders:
>  >  Wir nehmen an, die Funktion [mm]f: M \to \IR[/mm] habe genau
> eine
> > Maximalstelle [mm]m_0 \in M\,.[/mm] Wenn nun [mm]g: I \to \IR[/mm] streng
> > wächst ([mm]I \subseteq \IR[/mm] soll dabei den Wertebereich von
> > [mm]f\,[/mm] umfassen!), dann hat auch [mm]g \circ f: M \to \IR[/mm] genau
> > eine Maximalstelle, und zwar die gleiche wie [mm]f\,,[/mm] nämlich
> > [mm]m_0 \in M\,.[/mm]
>  >  
> > Wenn man nun fragt, wie denn auch der Wert [mm]f(m_0)[/mm] aussieht,
> > so kann man natürlich nicht [mm](g \circ f)(m_0)[/mm] mit [mm]f(m_0)[/mm]
> > identifizieren - sondern, wenn man [mm](g \circ f)(m_0)[/mm] kennt,
> > dann kann man zurückrechnen
>  >  [mm]f(m_0)=g^{-1}(g(f(m_0)))\,.[/mm]
>  >  
> > Okay?
>  >  
> > Hier war ja quasi [mm]f: \IR^2 \supseteq A \to \IR[/mm] nichts
> > anderes als die euklidische Standardnom eingeschränkt auf
> > irgendeine gewisse Teilmenge [mm]A\,[/mm] des [mm]\IR^2\,,[/mm] die aus der
> > Aufgabenstellung folgt.
>  >  Betrachten wir erstmal dieses [mm]f\,,[/mm] d.h. [mm]\IR^2 \supseteq A \in x=(x_1,x_2) \mapsto f(x)=f((x_1,x_2))=:f(x_1,x_2):=\|x\|_2=\sqrt{x_1^2+x_2^2}\,.[/mm]
>  
> >  

> > Okay? Wenn Du nun fragst "Wo wird der maximale Abstand auf
> > [mm]A\,[/mm] zum Nullpunkt [mm](0,0)[/mm] des [mm]\IR^2[/mm] angenommen?", dann kannst
> > Du versuchen, einen (oder mehrere) Punkt(e) von [mm]A\,[/mm] so zu
> > bestimmen, nennen wir solch' einen [mm]m_0\,,[/mm] so dass [mm]f\,[/mm] an
> > diesem [mm]m_0 \in A[/mm] maximal wird. Das kannst Du auch machen,
> > indem Du irgendeine streng monoton wachsende Funktion [mm]I \to \IR[/mm]
> > nach [mm]f\,[/mm] schaltest ([mm]I \subseteq \IR[/mm] soll den Wertebereich
> > von [mm]f\,[/mm] umfassen), d.h. Du betrachtest beispielsweise die
> > Quadratfunktion [mm]g: [0,\infty) \to \IR[/mm] (der
> > Definitionsbereich von [mm]g\,[/mm] sollte allerdings natürlich den
> > Wertebereich von [mm]f\,[/mm] umfassen!) und bestimmst bzgl. [mm]g \circ f[/mm]
> > dann "Maximalstelle(n)".
>  >  
> > Mathepower hat quasi vorgeschlagen, die streng monoton
> > wachsende Funktion [mm]g:[0,\infty) \to [0,\infty)[/mm]
> > "nachzuschalten", wobei das [mm]f\,[/mm] wie oben ist. Er sagt
> > dann:
>  >  Anstatt [mm]f\,[/mm] auf Maximalstellen in [mm]A\,[/mm] zu untersuchen,
> kann
> > man auch [mm]g \circ f=f^2[/mm] auf Maximalstellen untersuchen -
> > denn diese haben die gleichen MAXIMALSTELLEN in [mm]A\,.[/mm]
>  >  
> > Der Wert der Maximalstellen ist natürlich ein anderer:
> > Wenn [mm]m_0 \in A[/mm] eine Maximalstelle in [mm]A\,[/mm] für [mm]f\,[/mm] ist, so
> > nimmt [mm]f\,[/mm] dort den Wert [mm]f(m_0)[/mm] an, aber [mm]g \circ f[/mm]
> > natürlich den Wert [mm]g(f(m_0))\,.[/mm]
>  >  
> > Bei Dir also: Wenn [mm]m_0[/mm] eine Maximalstelle in [mm]A\,[/mm] bzgl. [mm]f \,[/mm]
> > ist, dann ist [mm]f(m_0)=\|m_0\|_2=\|m_0\|[/mm] "der Abstand von [mm]m_0[/mm]
> > zu [mm](0,0)[/mm]". Mit [mm](g \circ f)[/mm] (mit [mm]g(x)=x^2[/mm] für [mm]x \ge 0[/mm])
> > findest Du die gleiche Maximalstelle, aber der
> > Funktionswert [mm](g \circ f)(m_0)=g(f(m_0))=\|m_0\|^2[/mm] gibt
> > natürlich nicht den Abstand von [mm]m_0[/mm] zu [mm](0,0)[/mm] an, sondern
> > eben das Quadrat dieses Abstands. Daher musst Du, wenn Du
> > [mm]m_0[/mm] kennst, aber [mm]f(m_0)[/mm] berechnen willst, hier bei
> > [mm]g(f(m_0))[/mm] noch [mm]g^{-1}[/mm] nachschalten.
>  >  
> > Also das einzige, was Du Dir klar machen musst: Wenn ich
> > hier ein streng wachsende Funktion nach einer anderen
> > schalte, dann nimmt diese Verkettung an genau den gleichen
> > Stellen wie die Ausgangsfunktion ihre Maxima und Minima an
> > (und solche sind bei beiden gleichzeitig existent oder
> > nicht etc. pp.).
> >
> > Die ExtremSTELLEN sind gleich (und von gleicher Art).
>  >  
> > Die Verkettung hat aber natürlich andere FunktionsWERTE,
> > d.h. die ExtremWERTE sind verschieden - aber mit [mm]g\,[/mm] bzw.
> > [mm]g^{-1}[/mm] kann man sie einander in Bezug bringen.
>  >  
>
> Wow ! Da hast du dir echt viel Mühe gegeben. Habe gerade
> 20 min gebraucht um das nachzuvollziehen. Klingt allerdings
> alles sehr schlüssig und nachvollziehbar :) - gefällt.
> Solltest Lehrer/Dozent werden ;D (falls du das nicht schon
> bist)

nein, das bin ich nicht. So einfach wird man nicht Dozent ( außerdem sind die Gebiete, wo ich es werden "wollte", meist überlaufen - denn Analysis ist doch sehr beliebt ;-) ). Und als Lehrer wäre ich auch nicht so wirklich geeignet, glaube ich. In kleinen Gruppen habe ich schonmal Nachhilfe gegeben, bei mehr als vier Leuten - und das waren dann ja meist interessierte Menschen - wird's mir zu anstrengend. ;-)
  

> Danke für die ausführliche Erläuterung. Hätte nicht
> erwartet dass du dir so viel Zeit dafür nimmst :)
> Respekt.

Kein Ding!
  

> Hab auf jedenfall was gelernt. THX

Das freut mich - zumal viele Leute da auch schnell durcheinander kommen. Manchmal ist man da auch bei Optimierungsaufgaben verwirrt - zum Beispiel, wenn man einen maximalen Funktionswert einer Funktion berechnen soll (solch einer muss natürlich dann auch existieren!), der Übungsleiter das auch alles korrekt macht, aber dann aufhört, wenn er eine Stelle gefunden hat, an der dieser angenommen wird. Im Wesentlichen ist die Aufgabe dann ja auch komplett gelöst, denn Einsetzen und ausrechnen kann jeder alleine - aber da sollte dann auch noch irgendwo ein Hinweis kommen so nach dem Motto "Wir sind hier noch nicht ganz fertig, aber das Berechnen des maximalen Funktionswertes überlasse ich nun Ihnen!".

Denn manche hören dann eben an solchen Stellen in der Klausur einfach auf, weil sie "nach Schema" lernen bzw. sich nie den Unterschied zwischen "Extremstellen" und "Extremwerte" klar gemacht haben - noch verwirrter sind sie meist dann, wenn sie gar sowas wie einen "Extrempunkt" angeben sollen: Dieser besteht nämlich aus der Extremstelle und dem zugehörigen Extremwert, ist also ein Element des Graphen der betrachteten Funktion. Und manche Lehrbücher sprechen auch von Extrempunkten, wenn sie nur eine Extremstelle meinen. Also da muss man schon aufpassen und sich am besten immer vergewissern, was der Autor wie bezeichnet!

Gruß,
  Marcel

Bezug
                                        
Bezug
Maximaler Abstand: Antwort
Status: (Antwort) fertig Status 
Datum: 07:37 Mi 20.06.2012
Autor: angela.h.b.


> Dann käme ich auf
> [mm]L(x,y,\lambda)=x^2+y^2+\lambda(x^3+2x^2-2x+y^2)[/mm]
>  
> und die Ableitungen sind dann:
>  
> [mm]2x+\lambda(3x^2+4x-2)=0[/mm]
>  
> [mm]2y+2y\lambda[/mm] = 0
>  
> und
> [mm]x^3+2x^2-2x+y^2=0[/mm]

Hallo,

die Ableitungen sind das nicht.
Das ist schon das zu lösende GS.
(Ich weiß schon, was Du meinst, aber achte genauer auf das, was Du sagst.)

Die von Dir geplante Vorgehensweise klingt prinzipiell vernünftig,
aber mir jedenfalls ist die Aufgabenstellung nicht ganz klar.
Beachte dazu die Anmerkungen weiter unten im Thread.

LG Angela

>  
>
> Aus der 2. Gleichung folgt [mm]2y(1+\lambda)=0,[/mm] also ich y = 0
> oder [mm]\lambda[/mm] = -1. Für y = 0 kann ich dann [mm]x^3+2x^2-2x=0[/mm]
> lösen.
>  
> Für [mm]\lambda[/mm] = -1
>  Habe ich dann in der ersten Gleichung [mm]2x-3x^2+4x-2=0[/mm] was
> ich lösen kann. Und mit der Lösung und der Nebenbedingung
> kann ich dann y ausrechnen.
>  
> Wäre das dann die richtige Lösung?
>  
> Vielen Dank


Bezug
        
Bezug
Maximaler Abstand: Antwort
Status: (Antwort) fertig Status 
Datum: 23:49 Di 19.06.2012
Autor: SEcki


> ich soll den maximalen Abstand der Fläche
> [mm]x^3+2x^2-2x+y^2=0[/mm] vom Ursprung bestimmen.

Der ist offensichtlich unendlich. Trickfrage?

SEcki

Bezug
                
Bezug
Maximaler Abstand: Mitteilung
Status: (Mitteilung) Reaktion unnötig Status 
Datum: 00:26 Mi 20.06.2012
Autor: Marcel

Hallo Secki,

> > ich soll den maximalen Abstand der Fläche
> > [mm]x^3+2x^2-2x+y^2=0[/mm] vom Ursprung bestimmen.

vielleicht wurden Einschränkungen an [mm] $x\,$ [/mm] vorenthalten...

Ansonsten hast Du natürlich Recht, da man das schon "durch den Graphen von [mm] $f(x):=\sqrt{-x^3-2x^2+2x}$" [/mm] erkennen würde.

Gruß,
  Marcel

Bezug
                
Bezug
Maximaler Abstand: nur hilbert kann helfen
Status: (Mitteilung) Reaktion unnötig Status 
Datum: 07:31 Mi 20.06.2012
Autor: angela.h.b.


> > ich soll den maximalen Abstand der Fläche
> > [mm]x^3+2x^2-2x+y^2=0[/mm] vom Ursprung bestimmen.
>  
> Der ist offensichtlich unendlich. Trickfrage?
>  
> SEcki

Hallo,

ich fürchte fast, daß die Wiedergabe der Aufgabenstellung ziemlich mißglückt ist.
Bei obiger Aufforderung würde ich jedenfalls [mm] f(x,y,z):=x^2+y^2+z^2 [/mm] auf der Menge  [mm] M:=\{(x,y,z)\in \IR^3|x^3+2x^2-2x+y^2=0\} [/mm] optimieren.
Wofür man, wie Du schon sagst, nichts rechnen muß - völlig unabhängig von irgendwelchen Einschränkungen an x.

Was da aber genau untersucht werden soll, ob vielleicht [mm] M:=\{(x,y)\in \IR^2|x^3+2x^2-2x+y^2=0\} [/mm] oder (wahrscheinlicher) die durch [mm] M:=\{(x,y)\in \IR^2|x^3+2x^2-2x+y^2=0, x\ge 0\} [/mm] begrenzte Fläche oder noch was anderes muß uns wohl hilbert persönlich sagen.

LG Angela




Bezug
                        
Bezug
Maximaler Abstand: Frage (beantwortet)
Status: (Frage) beantwortet Status 
Datum: 08:52 Mi 20.06.2012
Autor: Laurent

Eigentlich sollte man den Maximalen Abstand einer Schnittfläche bestimmen, und zwar von [mm] x^3-y^2-z^2=0 [/mm] und [mm] (x-1)^2+y^2+2z^2=1. [/mm]

Ich habe jetzt einfach die beiden Gleichungen so ineinander eingesetzt, dass die resultierende Gleichung nur noch in Abhängigkeit von x und z war.
Ändert dies die Aufgabenstellung wesentlich?



Ich glaube die gleiche Aufgabe hatte ich auch =/

Bezug
                                
Bezug
Maximaler Abstand: Mitteilung
Status: (Mitteilung) Reaktion unnötig Status 
Datum: 09:41 Mi 20.06.2012
Autor: hilbert

Ja, genau die Aufgabe hatte ich auch. Also kann ich x durch die 2. Nebenbedingung doch mit 0 [mm] \le [/mm] x [mm] \le [/mm] 2 eingrenzen oder?

Bezug
                                
Bezug
Maximaler Abstand: Antwort
Status: (Antwort) fertig Status 
Datum: 10:43 Mi 20.06.2012
Autor: angela.h.b.


> Eigentlich sollte man den Maximalen Abstand einer
> Schnittfläche bestimmen, und zwar von [mm]x^3-y^2-z^2=0[/mm] und
> [mm](x-1)^2+y^2+2z^2=1.[/mm]

Hallo,

aha. Nun kennen wir auch mal die zu bearbeitende Aufgabe.
Man hat also zwei Flächen im Raum gegeben und soll sagen, welches der Punkt des Schnittes der beiden Flächen ist, welcher vom Ursprung den maximalen Abstand hat. Richtig?

>  
> Ich habe jetzt einfach die beiden Gleichungen so ineinander
> eingesetzt, dass die resultierende Gleichung nur noch in
> Abhängigkeit von x und z war.
>  Ändert dies die Aufgabenstellung wesentlich?

Nun, die Aufgabenstellung steht ja fest...
Ob Dein Tun zu dieser paßt, ist mir nicht ganz klar.
Kommt halt darauf an, wie Du dann weitermachst.
Jedenfalls ist die Menge der Punkte mit
[mm] $x^3+x^2-2x+z^2=0$ [/mm]
nicht der Schnitt der beiden gegebenen Flächen!
(In dieser Fläche liegt der z.B. Punkt (0,4,0), welcher jedoch in den beiden gegebenen Flächen nicht liegt.)

Die gestellte Aufgabe, so, wie ich sie oben formuliert habe, ist eine Extremwertaufgabe mit zwei Nebenbedingungen. Wenn Du die Lagrangefunktion aufstellst, hast Du zwei Lagrangemultiplikatoren.

LG Angela




Bezug
Ansicht: [ geschachtelt ] | ^ Forum "Funktionen"  | ^^ Alle Foren  | ^ Forenbaum  | Materialien


^ Seitenanfang ^
ev.vorhilfe.de
[ Startseite | Mitglieder | Impressum ]